Paradoja del colapso de la función de onda en un estado no físico

"Una medición siempre hace que el sistema salte a un estado propio de la variable dinámica que se está midiendo, el valor propio al que pertenece este estado propio es igual al resultado de la medición".

— PAM Dirac, Los principios de la mecánica cuántica

Este es uno de los postulados de la mecánica cuántica. Sin embargo, hay algunos casos en los que esta afirmación conduce a contradicciones.

Por ejemplo, sabemos que las funciones propias del operador de cantidad de movimiento (en 1D por simplicidad)

pags ^ = i X

son ondas planas:

ψ pags ( X ) = A mi i pags X /

Estas funciones propias no son normalizables y por lo tanto no son aceptables como estados físicos.

Si tratáramos de aplicar el postulado citado al operador cantidad de movimiento, incurriríamos por tanto en una contradicción: el sistema no puede saltar a un estado propio del operador cantidad de movimiento, porque tal estado propio no sería normalizable y por lo tanto no sería un estado físico.

Esta paradoja generalmente se descarta diciendo que esta línea de razonamiento se aplica a una medida ideal , que no se puede realizar en la práctica, y que para la medida no ideal la situación es diferente. Pero esta respuesta no me parece satisfactoria: aunque tiene sentido, no está claro cuál es la razón teórica por la que una medida ideal no es realizable.

Parece que solo hay dos posibles soluciones a esta paradoja:

  1. El postulado citado es incorrecto.
  2. El operador de cantidad de movimiento está algo mal definido: por ejemplo, tal vez no podamos simplemente tomar su dominio como el conjunto de todas las funciones suficientemente regulares (*) F L 2 ( R ) como solemos hacer. En este caso, tal vez sea posible dar una definición del operador de cantidad de movimiento que concuerde con el postulado citado.

¿Cuál es una posible solución a esta paradoja?

PD: En lo que a mí respecta, está perfectamente bien responder que la solución es que una medida ideal no es físicamente realizable en la práctica, pero solo si tal afirmación está respaldada con argumentos teóricos rigurosos que expliquen por qué es así . .

(*) En ocasiones, la condición que se impone es la absoluta continuidad de F , pero no sé si se puede relajar.


Actualizaciones

  • Preguntas y respuestas relacionadas:

- Medida de observables con espectro continuo: Estado del sistema a posteriori (sugerido por ACuriousMind). Después de un poco de discusión, el autor agregó un apéndice maravilloso que tal vez pueda considerarse como una respuesta a esta pregunta.

- Mecánica cuántica - posición de medida .

  • Artículos relacionados:

Encontré este artículo y este artículo (descarga gratuita) que tratan sobre este problema exacto, pero son bastante técnicos y todavía tengo que profundizar en ellos.

Probablemente habrías visto este , pero aún estoy publicando por casualidad ... de todos modos, la única diferencia entre tu pregunta y la del enlace es la de las funciones propias de posición e impulso.
@PrasadMani Sí, he visto esa pregunta, pero creo que la mía es un poco más específica: estoy interesado en una solución formal rigurosa de esta paradoja. Gracias de cualquier manera.
Eche un vistazo a esta pregunta y respuesta (¡posible duplicado!): el postulado que cita es para un observable con espectro discreto no degenerado y debe ser reemplazado por el axioma de Lüders-von Neumann en general.
@ACuriousMind Esa respuesta es realmente interesante, pero de alguna manera no es completamente satisfactoria porque no está claro qué es esto. d debería ser exactamente. Tal vez debería echar un vistazo más de cerca a los POVM y al formalismo de los operadores de Kraus, como sugiere el autor de la respuesta en uno de los comentarios. Los estudié en un curso de información cuántica, pero no tan a fondo. Al final, todo se reduce a una formalización matemática rigurosa del proceso de medición, y hasta donde yo sé, no hay una sola teoría aceptada...
@ACuriousMind PD Creo que podemos considerar que esta pregunta no es un duplicado de esa debido a la posibilidad de que el operador de impulso esté mal definido de alguna manera. Pero eso sí, la otra parte más o menos igual.
@valerio92: d depende exactamente de cómo se lleve a cabo la medición. Además, hay muchos más tipos de medidas que no se describen exactamente en el axioma de Lüders-von Neumann. Y de hecho, en la práctica es indudable que es imposible hacer exactamente una medición de Lüders-von Neumann, como en la práctica la ventana ( X d , X + d ) no va a tener bordes afilados.
no está claro cuál es la razón teórica por la que una medición ideal no es realizable" - Creo que es el caso de que la medición del momento de una partícula con una precisión arbitraria requiere un tiempo arbitrariamente largo.
"Tal vez hay dos postulados de Luders diferentes ..." Tal como lo veo, el postulado hace ambas cosas: generaliza la noción de colapso de la función de onda a a. colapso de estados mixtos, b. colapsar en cualquier subespacio, no solo en un estado propio.
@BruceGreetham Quiero decir que en las fuentes que encontré (y cité) no hay nada sobre la función de onda después de que la medición esté en la forma x yo ψ , dónde x yo es la función indicadora del intervalo α ± d , dónde α es el resultado y d el error experimental
Sí, lo entendí, lo que digo es que este es un caso especial del uso de proyectores (el postulado de la proyección). Para estados puros puedes proyectar el estado; para estados mixtos, usa el mismo proyector para proyectar la matriz de densidad ρ > PAGS ρ PAGS . Puedo intentar escribir para aclarar, pero tal vez estés buscando algo más matemáticamente riguroso de lo que puedo ofrecer.
Estoy 90% seguro de que esto es un duplicado, pero no puedo encontrar el original.
El punto de vista de un experimentador: la medida ideal no es medible ni siquiera teóricamente porque CUALQUIER medida implica potenciales de algún tipo. Los potenciales generan valores propios discretos. Para ver la partícula representada por la onda plana, debe interactuar y la interacción implica un potencial que tendrá funciones propias apropiadas para el problema en estudio.
@DanielSank Después de publicar este, encontré dos preguntas que son muy similares. Agregué enlaces a esas preguntas en la sección Actualizar.

Respuestas (3)

El asunto de todo es esto:

Hay dos tipos de funciones propias de los operadores hermitianos. Los que admiten espectros discretos (los autovalores están separados entre sí) y los otros, espectros continuos (los autovalores llenan un rango completo). Si los espectros son continuos, entonces NO representan posibles funciones de onda (solo una combinación lineal de ellas ... sí, un paquete de ondas gaussianas puede ser normalizable). En el caso de los operadores de cantidad de movimiento,

i F pags ( X ) X = pags F pags ( X )
.... F pags ( X ) es una función propia de impulso .... resolver lo anterior da
F pags ( X ) = A mi i pags X
que no es un cuadrado integrable.

Pero dado que el momento es un observable, solo tomamos valores reales de p y usamos la ortonormalidad de Dirac por

F pags ( X ) F pags ( pags ) d X = | A 2 | mi ( pags pags ) X d X = | A 2 | 2 π d ( pags pags )
, y luego seleccionando A = 1 2 π , tenemos
F pags | F pags = d ( pags pags )
......

Ahora, esto significa que las funciones propias del momento son sinusoidales (esto en sí mismo es irrealizable ya que cualquier onda sinusoidal VERDADERA o PERFECTA tiene que extenderse desde a )

Pero no existe tal cosa como una partícula con momento definido , cortesía del principio de incertidumbre de Heisenberg... también implica que la medición no puede colapsar una función de onda a un estado propio con un momento perfectamente definido.

Esta es la razón por la que creamos un paquete de ondas normalizable con un rango estrecho de momentos... para que todo sea físicamente realizable. Ninguna de las funciones propias de pags ^ viven en el espacio de hilbert, pero aquellos con valores propios reales (paquetes de ondas) y que son dirac normalizables sí lo hacen. Ellos (funciones propias de pags ^ ) no representan posibles estados físicos pero son muy útiles en problemas como la dispersión desde una colina potencial o una barrera.

Referencia: - Griffiths, Introducción a la mecánica cuántica


EDITAR

No vote a favor de mi respuesta, ya que no aborda por completo las inquietudes planteadas por OP (eche un vistazo a la sección de comentarios debajo de esta respuesta), es decir, un tratamiento formal del colapso de 'paquete de ondas' (no función de onda) ... i Lo siento mucho si invocar tal declaración es incorrecto. En el mejor de los casos, mi respuesta está parcialmente completa.

Sí, he leído el libro de Griffith. Básicamente, su respuesta es que el postulado citado no se aplica al operador de momento y que cuando medimos el momento en la vida real, obtenemos un paquete de ondas con un rango estrecho de momentos. Esta explicación podría ser correcta, pero no es completa: ¿cómo ocurre exactamente el colapso de tal paquete de ondas? ¿Cómo podemos tratarlo formalmente? Además, ¿no es posible que, para empezar, el problema esté en nuestra definición de operador de cantidad de movimiento?
En realidad, no creo que su aplicación excluya pags ^ ; si te molesta, solo haz la transformada de Fourier, ve al espacio de momento, terminarás básicamente con el mismo problema; con operador de posición dado por i X y el impulso como d ( pags pags ) La dispersión de la función de onda después del colapso surge naturalmente de la naturaleza de los operadores en la mecánica cuántica y cuando medimos X ^ en base a la posición, no pretendemos que esté en una posición X del estado propio d ( X X ) con precisión arbitraria; está dentro del límite de la incertidumbre de Heisenberg.
Ahora, por supuesto, podría continuar y preguntar (al menos yo lo haría), ¿qué sucede en el caso de los estados propios de energía? Dado que son normalizables en la mayoría de los casos (no partículas libres o similares), ¿tienen una dispersión en la energía medida después del colapso de la función de onda o deberíamos especificarlo mencionando también el TIEMPO que se tardó en medir esa energía y, por lo tanto, poner una estimación en la propagación de la energía? Este es el alcance de mi conocimiento, lo siento. ¡Pero espero respuestas más refinadas y mejores a su pregunta, así como a la de este comentario!
Dije erróneamente que el operador de posición en el espacio de momento es i X ..... leer eso como i pags
"Pero no existe tal cosa como una partícula con momento definido, cortesía del principio de incertidumbre de Heisenberg" El principio de incertidumbre de Heisenberg simplemente dice que no existe tal cosa como una partícula con posición definida y momento definido, no dice ni posición ni momento puede ser definitivo.
En realidad, dice que tanto la posición como el momento no pueden especificarse con precisión arbitraria; pero para la pregunta que OP publicó, la forma en que interpreté el principio no es defectuosa; solo que si mido el impulso de la partícula con precisión arbitraria, no tengo NINGUNA información de la posición de la partícula cuyo impulso acabo de medir, por lo tanto, es un poco difícil comprender de qué acabo de medir el impulso ya que la partícula desaparece el siguiente instante (o no) y no puedo identificar la ubicación de la partícula (encontrar el impulso debería haber resuelto eso) con una precisión sensible
Así que no significa absolutamente nada decir que la posición de la partícula es alguna X o el momento de la partícula es algo pags ......ya que la partícula puede estar en cualquier lugar debido a los límites de incertidumbre...lo cual no debería ser ya que medir la posición perfecta o el momento perfecto significa que sabemos exactamente dónde está o debería estar la partícula en el futuro, en cierto sentido.
Esta respuesta tiene las matemáticas correctas, pero siento que ignora por completo el simple hecho físico de que no se puede medir el pags ^ operador simplemente porque eso requeriría un laboratorio infinitamente grande.

Esta respuesta llega extremadamente tarde, pero creo que vale la pena publicar una respuesta física, incluso si solicitó una respuesta matemática.

Primero considere una medida de posición. Si hiciera una medición de posición ideal, la función de onda colapsaría a un estado propio de posición, que no es normalizable y, por lo tanto, no es un estado físico válido. La resolución es que las medidas de la posición ideal en realidad no existen, porque las medidas reales tienen una solución finita.

Como una forma aproximada de explicar esto, suponga que su medida solo puede devolver valores que son enteros, en algunas unidades. Entonces efectivamente estás midiendo al operador.

X ^ F = X ^
donde la función suelo de un operador se define por elementos: si X ^ | X = X | X , después
X ^ F | X = X | X .
Como resultado, todos los estados | X por X [ norte , norte + 1 ) son vectores propios degenerados de X ^ F . Todas las posiciones en este rango corresponden al mismo resultado de medición norte .

En el caso de la degeneración, el postulado del colapso está ligeramente generalizado. El espacio de Hilbert ahora se divide en subespacios ortogonales, uno para cada valor propio distinto del operador medido. Tras la medición de ese operador, el vector de estado colapsa en su proyección en uno de estos subespacios, con probabilidad proporcional a la magnitud al cuadrado de su proyección. (Esto se reduce al postulado que da Dirac cuando todos los subespacios son unidimensionales, es decir, sin degeneración).

Por lo tanto, si tiene un estado con función de onda ψ ( X ) y medir X ^ F , lo colapsas para

ψ ( X ) = ψ ( X ) { 1 norte X < norte + 1 0 de lo contrario
si el resultado de su medición es norte . Este estado es perfectamente normalizable.

La misma lógica se aplica exactamente a pags ^ , con la lógica anterior ocurriendo en el espacio de Fourier en lugar del espacio real, y lleva a la misma conclusión: obtienes un estado final normalizable con ancho finito en el espacio de Fourier, que corresponde a un paquete de ondas en el espacio real.

Así que todo sale bien... pero ¿es "riguroso"? Bueno, el objetivo de la mecánica cuántica es predecir resultados físicos. El propósito de formularlo en términos de postulados es solo darnos una base concreta para calcular los resultados, con el objetivo final de hacer coincidir el experimento. Es bien sabido que si tomas los postulados declarados de Dirac al pie de la letra y los usas para calcular cantidades no físicas, entonces puedes obtener todo tipo de contradicciones matemáticas: sus postulados son descuidados incluso para los estándares de los físicos. Continuamos enseñándolos porque funcionan para cualquier experimento que podamos realizar.

Si quisiera ser "riguroso", lo haría agregando montones de postulados adicionales que equivalen a decir "no se le permite usar X ^ en el postulado del colapso, pero cosas como X ^ F está bien". Pero a los físicos experimentales no les importará en absoluto, porque siempre han sabido que existen limitaciones en lo que pueden medir, postulados sofisticados o no.

Aquí creo que está la resolución de la paradoja primero recuerda que la cita que diste anteriormente corresponde a H mi r metro i t i a norte operadores. La idea clave es la siguiente:

El operador de cantidad de movimiento no es un operador hermitiano en el espacio de funciones en el que las ondas planas son miembros y pueden considerarse funciones propias del operador de cantidad de movimiento. En este dominio extendido, el operador de cantidad de movimiento no corresponde a ninguna medida.

Para que los operadores de cantidad de movimiento sean hermitianos, nos gustaría demostrar que ϕ PAGS ψ = PAGS ϕ ψ . Considere cómo probamos que el operador de cantidad de movimiento es hermitiano, hacemos el siguiente cálculo: ϕ ( i d d X ψ ) d X = i ( ϕ ψ | d d X ϕ ψ d X ) . Note que para el primer término en el lado derecho la función tiene que desaparecer en ± . Si esto sucede, entonces el operador de cantidad de movimiento es igual a su adjunto hermitiano y, por lo tanto, es hermitiano. Esto excluye las ondas planas porque no desaparecen en ± . Entonces, el operador de cantidad de movimiento no es hermético en el espacio de funciones en el que las ondas planas son miembros. Por tanto, el operador momento no corresponde a ninguna medida física en este espacio de funciones.

Creo que la resolución no tiene nada que ver con si la medida es ideal o no, si el operador de impulso está mal definido o no. El operador momento no corresponde a ninguna medida, sea ideal o no, si actúa sobre un dominio extendido que incluye ondas planas porque no es hermético sobre este dominio extendido.

Pero es precisamente porque las ondas planas no están contenidas en L 2 que podemos decir que la cantidad de movimiento es un operador hermitiano: en su cálculo, ni ψ ni ϕ puede ser una onda plana, porque no están contenidos en L 2 .
@valerio92 El punto es que dependíamos crucialmente del hecho de que estábamos tratando con la función en L 2 para concluir que el operador de cantidad de movimiento era hermitiano. Si el operador momento actúa sobre ondas planas o el espacio que incluye ondas planas, no es hermitiano y por lo tanto no puede corresponder a ninguna medida.
@valerio92 En el dominio de L 2 el operador de momento es hermitiano pero no en el dominio extendido que incluye ondas planas. Este es mi punto.
Sí, pero ¿quién dijo que debe ser ermitaño en un dominio tan extenso? Desde un punto de vista formal, será simplemente un operador sin función propia, porque la solución del problema de valores propios es una onda plana, que no está contenida en L 2 . [PS en realidad el dominio propio de pags es { ψ L 2 ( R ) : ψ L 2 ( R ) } , pero esto es solo un detalle en la presente discusión]
@ valerio92 debe ser hermitiano en el dominio extendido porque quiere hablar sobre medidas. Si desea hablar sobre la medición, la mecánica cuántica dice que debe tener un operador hermitiano y luego sigue la afirmación de Dirac. Pero en el dominio extendido no tiene un operador hermitiano, todo lo que tiene es un operador que tiene ondas planas como funciones propias. Eso no es lo suficientemente bueno
El hecho de que sea un operador sin funciones propias en el dominio de interés, L 2 no plantea preocupaciones ya que, en general, los operadores ilimitados en espacios de Hilbert de dimensión infinita no tienen que tener funciones propias.